To cut costs, a high school modified its air-conditioning system to increase its efficiency. The modified system, how...

jrowland on December 30, 2018

Please explain

Can you explain this one? I chose D.

Replies
Create a free account to read and take part in forum discussions.

Already have an account? log in

Ravi on January 1, 2019

@jrowland,

Happy to help! This is a strengthen with a necessary premise question.
The stimulus tells us that the high school modified its air
conditioning system to increase its efficiency, which would help the
school cut costs. The resulting modified system then caused the
humidity in the school to decrease by 18 percent.

Then, 24 hours after the decrease in air humidity, a 25 percent
increase in the number of visits to the school nurse was reported.

The argument then concludes that a decrease in humidity can make
people ill. But, wait a second. We just know that there was a 25
percent increase in visits to the school nurse. Do we know these
visits were for an illness? We don't. This is a huge hole in this
argument.

As mentioned above, this is a strengthen with a necessary premise
question, so we need to find an answer that, if negated, makes our
argument fall apart. In other words, the answer choice will be
something that must be true in order for our argument to stand a
chance.

Answer A says that at least some of the visits to the school nurse
after the system was modified were due to illness. The negation of
this is that none of the visits to the school nurse after the system
was modified were due to illness. If the negation were true, then our
argument would fall apart because NONE of the visits to the school
nurse after the system was modified were for illness. This destroys
the argument's conclusion that a decrease in humidity can make people
ill because it casts serious doubt on the support used (that after the
decrease in humidity, there was a 25 percent increase in visits to the
school nurse). But with A's negation, we know that none of the visits
to the nurse were for an illness, so how could the decrease in
humidity make people ill?

Answer D says that a decrease of 18 percent in air humidity causes an
increase of 25 percent in one's probability of becoming ill. The
negation of this is that a decrease of 18 percent in air humidity
doesn't necessarily cause an increase of 25 percent in one's
probability of becoming ill. This means that maybe sometimes it causes
a 24 percent (or less) increase, and other times it could cause a 26
percent (or more increase) in one's probability of becoming ill.

Does this wreck the argument? Absolutely not. The negation still
leaves open the possibility that these additional visits to the nurse
could have been from humidity. I'm suspecting you chose this answer
because it contained some of the same numbers (18 percent and 25
percent) that we saw in the stimulus, but the rest of the content of
this answer allows us to eliminate it.

Does this make sense? Let us know if you have any other questions!

Abigail-Okereke on December 5, 2021

Can you explain why the rest are incorrect also? I choose B and I don't know why its incorrect.

Jay-Etter on January 26, 2022

Hi @Abigail-Okereke,
Sure. So remember that for strengthen with necessary premise the key is that the correct answer option must be required for the argument to function. To check this, we use the negation test (which Ravi explained above).

B says "Most of the students at the high school suffered from the decrease in air humidity". The conclusion of our argument is that a decrease in humidity can make people ill. So do we care about whether or not the majority of the students suffered from this? Not really.

If we negate B "Not most (I.e. 49% or less) of the students suffered from the decrease in air humidity", this doesn't do anything to our conclusion which is just that humidity decrease CAN make people ill. The conclusion wasn't that it makes MOST people ill. Therefore the negation of B doesn't ruin the argument.

**A general tip: For strengthen with necessary premise questions, options with MOST as a quantifier statement are generally not good options. This is because Most can mean 51% and not most can mean 49%, so it is rare that flipping from 51% to 49% (the negation test) will ruin the argument. The only times that a MOST quantifier statement will be a good option for necessary assumption questions is when the conclusion of the argument itself is concerning whether something is/isn't a majority.